AHZolfaghari

Well-Known Member
ارسال ها
935
لایک ها
1,654
امتیاز
93
#61
پاسخ : ماراتن نظریه اعداد (سطح مقدماتی)

فرض مي كنيم:
ثابت مي كنيم توان هر عدد اولي كه مخرج رو ميشماره از توان همون عدد توي صورت كمتره. داريم:

پس كافي است براي هر
ثابت كنيم:

اگر
در اين صورت حكم ثابت شده است پس فرض كنيد i اي موجود باشه كه اين رابطه بر قرار نباشه در اين صورت مي دانيم كه توان
در كسر
برابر با
و در كسر
برابر با
است كه حاصل ضرب اين دو توان صفر را به ما مي دهد حالا كافي است توان
در عبارت
را مورد بررسي قرار بدهيم كه در اين عبارت هم حكم بر قرار مي بأشد پس در كل حكم ثابت مي شود.
یه راه حل ساده : از قضیه بزو استفاده کنید.





این هم بدیهی است که طبیعی هستش !!!!!!!!!!
حالا بقیه سوال بذارن !!!​
 

math1998

New Member
ارسال ها
336
لایک ها
224
امتیاز
0
#62
پاسخ : ماراتن نظریه اعداد (سطح مقدماتی)

یه راه حل ساده : از قضیه بزو استفاده کنید.





این هم بدیهی است که طبیعی هستش !!!!!!!!!!
حالا بقیه سوال بذارن !!!​
خودت ادامه بده سوالات خیلی خوبی میذاری .

---- دو نوشته به هم متصل شده است ----

تمام اعداد طیبیعی
را بیابید که
طبیعی باشد .
 

m-saghaei

New Member
ارسال ها
338
لایک ها
258
امتیاز
0
#63
پاسخ : ماراتن نظریه اعداد (سطح مقدماتی)

از آقای ذوالفقاری به خاطر سوالای خوبشون تشکر میکنم.واقعا به جرئت میشه گفت ایشون غول المپیاده ! جدا میگم !
 

AHZolfaghari

Well-Known Member
ارسال ها
935
لایک ها
1,654
امتیاز
93
#64
پاسخ : ماراتن نظریه اعداد (سطح مقدماتی)

خودت ادامه بده سوالات خیلی خوبی میذاری .

---- دو نوشته به هم متصل شده است ----

تمام اعداد طیبیعی
را بیابید که
طبیعی باشد .
کوچک ترین عامل اول n رو برابر q در نظر میگیریم . بوضوح داریم :

حالا فرض کنید مرتبه 4 به پیمانه n باشه d پس d یکی از مقسوم علیه های n هستش پس از q بزرگ تر هستش . اما بنابر قضیه فرما و اینکه
بدست میاد که
که تناقض داره پس d لاجرا باید یا خود q بشه یا برابر با یک باشه خود q که نیست ( خیلی بدیهیه ) پس d=1 پس
پس


پس کوچک ترین عامل اولش شد 3



با نوشتن لم دوخط بدست میاد که k بر سه بخش پذیر نیست .
حالا p رو کوچک ترین عامل اول k یا به عبارتی دیگر کوچک ترین عامل اول n بعد از سه میگیریم.
بازم مثل حالت قبل میشه

پس مرتبش رو میگیریم

حالا
خودش یکی از مقسوم علیه های n هستش و اینکه
از p کمتره پس
عامل اول جدید نیست . پس یا یک توانی از سه هستش یا یک هستش یا خود سه.
توانی از سه که نمیتونه باشه چون اثبات کردیم که n حداکثر یه عامل سه داره . پس یا یک هستش یا سه
اگه یک باشه p مساوی سه میشه که تناقضه چون فرض این بوده که p از سه بزرگتره پس
برابر سه میشه
پس

پس

پس n بر 7 بخش پذیر هستش.
حالا :

پس تو هر مرحله میتونیم توان 3 رو سه تا کم کنیم و در میرسه به باقی مانده n بر سه که یعنی به 3 میرسه یعنی 7 عاد میکنه 3 رو که تناقضه پسn عامل اول دیگه ای بجز 3 نخواهد داشت و چون اثبات کردیم این عوامل از یک بیشتر نیست پس جواب فقط n= 3 , n=1 می باشد.

سوال 3 المپیاد جهانی 1990 - پکن ، چین ( سوال خیلی قشنگی بود )
 

AHZolfaghari

Well-Known Member
ارسال ها
935
لایک ها
1,654
امتیاز
93
#65
پاسخ : ماراتن نظریه اعداد (سطح مقدماتی)

خودت ادامه بده سوالات خیلی خوبی میذاری .

---- دو نوشته به هم متصل شده است ----

تمام اعداد طیبیعی
را بیابید که
طبیعی باشد .
کوچک ترین عامل اول n رو برابر q در نظر میگیریم . بوضوح داریم :

حالا فرض کنید مرتبه 4 به پیمانه n باشه d پس d یکی از مقسوم علیه های n هستش پس از q بزرگ تر هستش . اما بنابر قضیه فرما و اینکه
بدست میاد که
که تناقض داره پس d لاجرا باید یا خود q بشه یا برابر با یک باشه خود q که نیست ( خیلی بدیهیه ) پس d=1 پس
پس


پس کوچک ترین عامل اولش شد 3



با نوشتن لم دوخط بدست میاد که k بر سه بخش پذیر نیست .
حالا p رو کوچک ترین عامل اول k یا به عبارتی دیگر کوچک ترین عامل اول n بعد از سه میگیریم.
بازم مثل حالت قبل میشه

پس مرتبش رو میگیریم

حالا
خودش یکی از مقسوم علیه های n هستش و اینکه
از p کمتره پس
عامل اول جدید نیست . پس یا یک توانی از سه هستش یا یک هستش یا خود سه.
توانی از سه که نمیتونه باشه چون اثبات کردیم که n حداکثر یه عامل سه داره . پس یا یک هستش یا سه
اگه یک باشه p مساوی سه میشه که تناقضه چون فرض این بوده که p از سه بزرگتره پس
برابر سه میشه
پس

پس

پس n بر 7 بخش پذیر هستش.
حالا :

پس تو هر مرحله میتونیم توان 3 رو سه تا کم کنیم و در میرسه به باقی مانده n بر سه که یعنی به 3 میرسه یعنی 7 عاد میکنه 3 رو که تناقضه پسn عامل اول دیگه ای بجز 3 نخواهد داشت و چون اثبات کردیم این عوامل از یک بیشتر نیست پس جواب فقط n= 3 , n=1 می باشد.

سوال 3 المپیاد جهانی 1990 - پکن ، چین ( سوال خیلی قشنگی بود )

---- دو نوشته به هم متصل شده است ----

از آقای ذوالفقاری به خاطر سوالای خوبشون تشکر میکنم.واقعا به جرئت میشه گفت ایشون غول المپیاده ! جدا میگم !
دیگه من انقدر ها هم تعریف کردنی نیستم . شما لطف داری دوست عزیز !
 

m-saghaei

New Member
ارسال ها
338
لایک ها
258
امتیاز
0
#66
پاسخ : ماراتن نظریه اعداد (سطح مقدماتی)

سلام
1- m و n دو عدد طبیعین که اختلافشون بیشتر از یکه. و m+n|4mn+1 ثابت کنید ب.م.م 2n-1 و 2m+1 برابر با یکه.
 

AHZolfaghari

Well-Known Member
ارسال ها
935
لایک ها
1,654
امتیاز
93
#68
پاسخ : ماراتن نظریه اعداد (سطح مقدماتی)

سلام
1- m و n دو عدد طبیعین که اختلافشون بیشتر از یکه. و m+n|4mn+1 ثابت کنید ب.م.م 2n-1 و 2m+1 برابر با یکه.
صورت درست سوال اینه :


اثباتش هم به این شکل هستش :

براحتی میتونیم نتیجه بگیریم که :




به طرز مشابه :








 

darya.f

New Member
ارسال ها
182
لایک ها
114
امتیاز
0
#69
پاسخ : ماراتن نظریه اعداد (سطح مقدماتی)

ثابت کنىد ببنهاىت عدد طبىعى n وجود دارد که
 

AHZolfaghari

Well-Known Member
ارسال ها
935
لایک ها
1,654
امتیاز
93
#70
پاسخ : ماراتن نظریه اعداد (سطح مقدماتی)

ثابت کنىد ببنهاىت عدد طبىعى n وجود دارد که
این استدلالی میخوام انجام بدم شاید زیاد مناسب نباشه اما من اول اومدم N های کوچیک رو چک کردم و به عدد خاصی نرسیدم.
بعد در پی این اومدم که
چند تا عامل اول گنده داشته باشه . مثلا اومدم n رو به پیمانه 17 گرفتم 6 و به پیمانه 7 گرفتم 4 .
بعد بدست اومد که n به پیمانه 119 میشه 74
بعد که چک کردم به ازای n=74 درست در میاد.
بعد دنباله
نوشتم و بعد دو تا از اعدادشو گرفتم ( ترجیحا عامل اول بزرگ داشته باشند ) بعد اومدم همین کارو کردم مثلاn=101 رو بدست اوردم.
به این ترتیب میشه n هایی رو دراورد به این شکل .
اما کلا باید یه کاری کنیم که
رو بشه تجزیه به دو تا عدد که نسبتا به هم نزدیک هستند تجزیه کرد
 

m-saghaei

New Member
ارسال ها
338
لایک ها
258
امتیاز
0
#71
پاسخ : ماراتن نظریه اعداد (سطح مقدماتی)

این استدلالی میخوام انجام بدم شاید زیاد مناسب نباشه اما من اول اومدم N های کوچیک رو چک کردم و به عدد خاصی نرسیدم.
بعد در پی این اومدم که
چند تا عامل اول گنده داشته باشه . مثلا اومدم n رو به پیمانه 17 گرفتم 6 و به پیمانه 7 گرفتم 4 .
بعد بدست اومد که n به پیمانه 119 میشه 74
بعد که چک کردم به ازای n=74 درست در میاد.
بعد دنباله
نوشتم و بعد دو تا از اعدادشو گرفتم ( ترجیحا عامل اول بزرگ داشته باشند ) بعد اومدم همین کارو کردم مثلاn=101 رو بدست اوردم.
به این ترتیب میشه n هایی رو دراورد به این شکل .
اما کلا باید یه کاری کنیم که
رو بشه تجزیه به دو تا عدد که نسبتا به هم نزدیک هستند تجزیه کرد
.
.
.
الان راه حل تموم شده یا نه؟

---- دو نوشته به هم متصل شده است ----

این استدلالی میخوام انجام بدم شاید زیاد مناسب نباشه اما من اول اومدم N های کوچیک رو چک کردم و به عدد خاصی نرسیدم.
بعد در پی این اومدم که
چند تا عامل اول گنده داشته باشه . مثلا اومدم n رو به پیمانه 17 گرفتم 6 و به پیمانه 7 گرفتم 4 .
بعد بدست اومد که n به پیمانه 119 میشه 74
بعد که چک کردم به ازای n=74 درست در میاد.
بعد دنباله
نوشتم و بعد دو تا از اعدادشو گرفتم ( ترجیحا عامل اول بزرگ داشته باشند ) بعد اومدم همین کارو کردم مثلاn=101 رو بدست اوردم.
به این ترتیب میشه n هایی رو دراورد به این شکل .
اما کلا باید یه کاری کنیم که
رو بشه تجزیه به دو تا عدد که نسبتا به هم نزدیک هستند تجزیه کرد
.
.
.
الان راه حل تموم شده یا نه؟
 

darya.f

New Member
ارسال ها
182
لایک ها
114
امتیاز
0
#72
پاسخ : ماراتن نظریه اعداد (سطح مقدماتی)

من فکر کردم مثلا جاى n ىه فرمى مثلا km ىا چىزاى اىن شکلى بزارىم که عبارت تجزىه شه,مثلا اگه به جا 2- تو سوال 1 بود 2m جواب مىداد....
 

AHZolfaghari

Well-Known Member
ارسال ها
935
لایک ها
1,654
امتیاز
93
#73
پاسخ : ماراتن نظریه اعداد (سطح مقدماتی)

من فکر کردم مثلا جاى n ىه فرمى مثلا km ىا چىزاى اىن شکلى بزارىم که عبارت تجزىه شه,مثلا اگه به جا 2- تو سوال 1 بود 2m جواب مىداد....
بله منم اون اول به سمت این کار رفتم اما نشد . شما به نتیجه ای نرسیدید ؟
 

ash1374

New Member
ارسال ها
253
لایک ها
422
امتیاز
0
#74
پاسخ : ماراتن نظریه اعداد (سطح مقدماتی)

راهنمایی:

سوالی مهم تر از راهنمایی بالا: فکر می کنید این عبارت چه جوری به ذهنم رسید؟
 

darya.f

New Member
ارسال ها
182
لایک ها
114
امتیاز
0
#75
پاسخ : ماراتن نظریه اعداد (سطح مقدماتی)

بله منم اون اول به سمت این کار رفتم اما نشد . شما به نتیجه ای نرسیدید ؟
راستش نتىجه خاصى نه ولى نظرم رو
هاست ولى بررسى نکردم
 

AHZolfaghari

Well-Known Member
ارسال ها
935
لایک ها
1,654
امتیاز
93
#76
پاسخ : ماراتن نظریه اعداد (سطح مقدماتی)

راهنمایی:

سوالی مهم تر از راهنمایی بالا: فکر می کنید این عبارت چه جوری به ذهنم رسید؟
اگه اشتباه نکرده باشم شما در پی این بودید که یه دنباله ای از اعداد بر حسب n بیابید که اولا صدق کنند دوما هر جمله بر حسب جمله قبلیش بدست بیاد مثلا ما از 74 میتونیم به 5400 برسیم
 

ash1374

New Member
ارسال ها
253
لایک ها
422
امتیاز
0
#77
پاسخ : ماراتن نظریه اعداد (سطح مقدماتی)

اگه اشتباه نکرده باشم شما در پی این بودید که یه دنباله ای از اعداد بر حسب n بیابید که اولا صدق کنند دوما هر جمله بر حسب جمله قبلیش بدست بیاد مثلا ما از 74 میتونیم به 5400 برسیم
خوب آره هدف که همون تجزیه شدن اون عبارت بود. سوال اینه که چطوری یه عبارتی بدست آوردم که تجزیه بشه؟ طبیعتاً آزمون و خطا نکردم! یعنی اگه یه سوال دیگه شبیه این دیدیم چه کار کنیم. یک مثال میزنم: نشان دهید بی شمار
طبیعی وجود دارد که
عامل اولی بزرگتر از
ندارد.
 
آخرین ویرایش توسط مدیر

REZA 73

Active Member
ارسال ها
139
لایک ها
184
امتیاز
43
#78
پاسخ : ماراتن نظریه اعداد (سطح مقدماتی)

خوب آره هدف که همون تجزیه شدن اون عبارت بود. سوال اینه که چطوری یه عبارتی بدست آوردم که تجزیه بشه؟ طبیعتاً آزمون و خطا نکردم! یعنی اگه یه سوال دیگه شبیه این دیدیم چه کار کنیم. یک مثال میزنم: نشان دهید بی شمار
طبیعی وجود دارد که
عامل اولی بزرگتر از
ندارد.
اگر n به صورت
باشه در شرط مساله صدق میکنه
 

math1998

New Member
ارسال ها
336
لایک ها
224
امتیاز
0
#79
پاسخ : ماراتن نظریه اعداد (سطح مقدماتی)

راهنمایی:

سوالی مهم تر از راهنمایی بالا: فکر می کنید این عبارت چه جوری به ذهنم رسید؟
ببخشید نمیفهمم حالا باید چک کنیم که
به نظرم اینم به اندازه حل صورت مسئله سخته به دلیل اینکه برای
های بزرگ هیچ چیز خوبی بهمون نمیده و کمکی نمیکنه ممکنه بیشتر توضیح بدید .
در ضمن خواهشا بگید اینا چطور به ذهنتون میرسه مثلا اون اتحادی که توی نظریه ممتاز گذاشته بودید امکان به ذهن رسیدنش وجود نداره از روش خاصی استفاده میکنید ؟!!!
 
آخرین ویرایش توسط مدیر

REZA 73

Active Member
ارسال ها
139
لایک ها
184
امتیاز
43
#80
پاسخ : ماراتن نظریه اعداد (سطح مقدماتی)

پس : n به توان دو منهای 2 برابر است با:
حال برای این که رابطه بالا برقرار باشه کافیه s به هنگ 7 برابر 1 باشه.در اون صورت چون دو پرانتز سمت چپ عاد کردن متباین هستند رابطه درسته.

امیدوارم تو محاسبات اشتباه نکرده باشم.
 
بالا